What is the Riemann Hypothesis?

In summary: X_p$ events. This is a deterministic approach and so the number of events is not sensitive to the choice of $n$. This estimation is done efficiently using the counting function $C(x)$ which is defined as$$C(x) = \frac{1}{x}\sum_{p \in \mathbb N} \left ( x^{p} - 1 \right )$$Theorem :There exists a function $f : \mathbb N \to \mathbb R$ such that $C(x) \propto f(x) \prod_{p
  • #1
mathbalarka
456
0
Throughout this note, I'll give a brief, explanatory, informal introduction to the Riemann Hypothesis (RH) explaining the statement of the conjecture, the difficulties of approaching it as well as some notable consequences of RH in the field of number theory. I hope the readers will enjoy the brief survey and would get inspiration to read further from this note. As the author (which is me) himself is an amateur and explains RH through his very limited understanding, he will appreciate any feedback from the readers. Corrections and suggestions for improvements are more than welcome.
 
Last edited by a moderator:
Physics news on Phys.org
  • #2
[The Riemann $\zeta$]

The central object of this conjecture is a special function, called the $\zeta$ function. It is defined as the sum

$$\zeta(s) = \sum_{n = 1}^\infty \frac1{n^s}$$

Exercise : Prove that this series is divergent for $\Re \leq 1$, so that the zeta function is defined only on the domain $\Re > 1$.

Much of this function's real and complex analytic properties are described in the integration tutorial of the calculus forum[1]. However, this function is also of number theoretic interest and is connected to primes. This is explained rather elementarily from the Euler product form :

$$\zeta(s) = \prod_{p} \left ( 1 - \frac1{p^s} \right )^{-1}$$

Where the product is taken over all the primes $p$. This is proved by noting that $\displaystyle \zeta(s) \prod_p \left ( 1 - \frac1{p^s}\right) = \sum_{n \in \Bbb N \setminus (p\#) \Bbb N} \frac1{n^s}$ where $p\#$ is the product $1 \cdot 2 \cdot 3 \cdots p$, often called the priomrial of $p$. According to the fundamental theorem of arithmetic, every natural number can be factorized into prime elements upto multiplicity. Thus, if there are finitely many primes, it is possible to continue the above process to reach some prime which would be the largest of the primes. In this case, without loss of generality, $p$ be that prime. Thus the last sum evaluates to $1$ as $\Bbb N \setminus (p \#) \Bbb N = \varnothing$, hence we are done. On the other hand, if there are infinitely many primes then one can continue this process forever, ending up with an infinite product instead. In both cases, the product holds.

Excercise : Verify the above proof by proving that the product $\prod_p \left ( 1 - \frac1{p^s} \right)$ converges for $\Re > 1$, under the hypothesis of infinitude of primes. If possible, make use of the convergence of $\zeta(s)$ in the domain of interest.

The above relation is definitely of interest. In fact, we'll shortly see that fiddling with this relation how quickly one arrives at the whole theory of distribution of primes. The trick, however, is to get our hands dirty and look at the edges of the domain of $\zeta$ where the function behaves not-so-smoothly.

$\zeta(s)$ at $s = 1$ diverges. This, as almost all of our readers, who once went through a course of calculus, can see, is the so-called harmonic series $1 + 1/2 + 1/3 + 1/4 + 1/5 + \cdots$. Now let us consider the product formula we have just derived, and let $s$ tend to $1$ from the right.

$$\lim_{s \to 1^{+}}\zeta(s) = \lim_{s \to 1^{+}} \prod_{p} \left ( 1 - \frac1{p^s} \right )^{-1} = \prod_{p} \left ( 1 - \frac1{p} \right)$$

However, the limit on the left side diverges. If we assume that there are finitely many primes, this can never happen, as the product on the right side would then have finitely many terms and a finite value. This is a contradiction, thus there are infinitely many primes.

Excercise : Produce a proof of infinitude of primes by considering $\zeta(s)$ and the product formula for $s = 2$, under the hypothesis of irrationality of $\pi^2$ [You are given $\zeta(2) = \pi^2/6$. For a proof, see[2] - there are six of them, choose whatever you like!] (Another interesting proof is mentioned at mathstackexchange[3])

This seems to be an interesting way to obtain the infinitude. Is it possible that if one let's $s$ wander around more singularities of $\zeta(s)$ then more information about primes would be revealed? What if, say, we observe the nature of this function at the very boundary of it's domain, namely $\Re = 1$? Of course we are just hypothesizing here, but it turns out that $\Re = 1$ is indeed the right place to look for when it comes to the connection with primes.
 
Last edited:
  • #3
[The Prime Number Theorem]

Define $\pi(x)$ to be the number of primes upto some real $x$. The question of estimating $\pi(x)$ efficiently has always been an object of study in elementary analytic number theory. Let's try a probabilistic approach to have plausible guess for a good estimate.

Pick a random integer $n$ from the interval $(1, x)$. The probability of $n$ being a prime is $P(X) = \pi(x)/x$. $X_p$ be the events of $n$ being divisible by some prime $p \leq \sqrt{x}$. Assuming that these events are independent,

$$P(X) = P \bigg(\bigcap X_p \bigg) = \prod_{p \leq \sqrt{x}} P(X_p) = \prod_{p \leq \sqrt{x}} \left ( 1 - \frac1{p} \right)$$

The last product looks like $\prod_p ( 1 - 1/p) = 1/\zeta(1)$, so one should expect that the product grows as $\ll 1/H_{\sqrt{x}}$, where $H_k$ is the $k$-th partial sum of the harmonic series. But then $H_k \sim \log(k)$ by comparing with the integral, thus $\prod_{p \leq \sqrt{x}} (1 - 1/p) \sim c/\log(x)$ for some constant $c$. It turns out with some computational effort that $c = 1$ is a good choice here, which implies the expected estimate $\pi(x) \sim x/\log(x)$.

Excercise : $\pi_2(x) = \#\{p \leq x : p, p+2 \in \mathcal{P}\}$ be the analogous twin prime counting function. Derive, on probabilistic grounds, that $\pi_2(x) \sim 2C_2 \cdot x/\log^2(x)$ where $C_2 = \prod_q (1 - (q-1)^{-2}) $, the product running through odd primes. [whether $\pi_2(x) \to \infty$ as $x \to \infty$ is yet an open problem in number theory]

$x$​
$\pi(x)$[4][5]
$\approx x/\log(x)$​
$10^2$​
$25$​
$21.71$​
$10^3$​
$168$​
$144.76$​
$10^4$​
$1229$​
$1085.73$​
$10^6$​
$78498$​
$72382.41$​
$10^8$​
$5761455$​
$5428681.02$​
$10^{12}$​
$37607912018$​
$36191206825.27$​
$10^{16}$​
$279238341033925$​
$271434051189532.39$​
$10^{17}$​
$2623557157654233$​
$2554673422960304.86$​
$10^{18}$​
$24739954287740860$​
$24127471216847323.75$​
$10^{19}$​
$234057667276344607$​
$228576043106974646.13$​
$10^{20}$​
$2220819602560918840$​
$2171472409516259138.25$​
$10^{21}$​
$21127269486018731928$​
$20680689614440563221.48$​
$10^{22}$​
$201467286689315906290$​
$197406582683296285295.96$​
$4 \cdot 10^{22}$​
$783964159847056303858$​
$768592742555118350978.96$​

This is the celebrated conjecture of Gauss now known as prime number theorem (PNT). Almost a 100 years later de la Vallee-Poussin and Hadamard came up with proofs of this phenomenon, both using an observation related to zeta function. Only once I have tried to demonstrate this proof from scratch, which took me something like 5-6 hours or so. I wouldn't try to sketch the whole argument here since I am not mental, though I'd produce a general idea involved behind the scene. Readers who are interested in a complete proof are suggested to read[6][7].

We have suspected in #2 that inspecting the zeta function at $\Re = 1$ might give some fruitful results. The precise result that we need (we'll see why) here is that $\zeta(s)$ has no zero at that line. Or, as in that line the zeta is not defined, more precisely, $\zeta(\sigma + it)$ never goes to $0$ as $\sigma \to 1^+$. This is proved (a la de la Vallee-Poussin) like this : Begin with the Euler product and start manipulating, the goal being to transform it into a function which doesn't take $0$ anywhere in the complex plane. An example is $\exp(z)$, which is the most convenient function we can go for.

$$\zeta(s) = \exp\left (\log \prod_p \left ( 1 - \frac1{p}\right) \right ) = \exp\sum_p \log\left(1 - \frac1{p}\right) = \exp\sum_p\sum_{n \geq 1} \frac{1}{np^{ns}} = \exp\sum_p\sum_{n \geq 1} \frac{\cos(nt\log p)}{np^{n\sigma}}$$

Recall that $s = \sigma + it$. The standard approach from here is to note that $3 + 4\cos(\theta) + \cos(2\theta) = 2(1 + \cos(\theta))^2 \geq 0$ and hence

$$\zeta(\sigma)^3 \cdot |\zeta(\sigma + it)|^4 \cdot |\zeta(\sigma + 2it)| = \exp\sum_p\sum_{n \geq 1} \frac{3 + 4\cos(nt\log(p)) + \cos(2nt\log(p))}{np^{n\sigma}} \geq 1$$

As $\zeta(s)$ has a pole at $s = 1$, $\zeta(\sigma) = O((\sigma - 1)^{-1})$ as $\sigma \to 1^+$; if $1 + it$ is a zero of $\zeta$ that'd mean $\zeta(\sigma + it) = O(\sigma-1)$. The last term is $O(1)$. Thus, $\zeta(\sigma)^3 \cdot |\zeta(\sigma + it)|^4 \cdot |\zeta(\sigma + 2it)| = O(\sigma - 1)$ but that is not possible from out inequality, which is our desired contradiction.

The connection with PNT is yet to be explained, however.

Theorem : The prime number theorem is equivalent to the equality $\displaystyle \sum_{n \geq 1} \frac{\mu(n)}{n} = 0$

For those who don't know, $\mu(n)$ is the Moebius function, defined to be $1$ when $n$ is square-free and has even number of factors, $-1$ when square-free and has odd number of factors and $0$ otherwise.

This is a very interesting equivalence. From the definition of $\mu(n)$ along with the Euler product of $\zeta(s)$, it is almost immediate that the equality holds for $\Re > 1$. The nontrivial part is to ensure this equality for $\Re = 1$, which is sufficient to prove the above version of PNT.

Delicate results of complex analysis are needed here to go through the whole proof in full rigor. Ideally, one uses the Perron's formula to obtain

$$\sum_{n < x} \frac{\mu(n)}{n^s} = \frac1{2\pi i} \int_{c - iT}^{c + iT} \frac1{\zeta(s+\omega)} \frac{x^\omega}{\omega} d\omega + O\left ( \frac{x^c}{Tc}\right) + O\left ( \frac{\log x}{T} \right)$$

For $s = 1 + it$, some $c > 0$ and some sufficiently large real $T$. One moves the the contour to the dent $[c - iT, -\delta - iT], [-\delta - iT, -\delta + iT], [-\delta - iT, c + iT]$ for some small $\delta > 0$ for which $\zeta(s + \omega)$ has no zeros for $\Re[\omega] \geq - \delta$; picking up the residue at $\omega = 0$ and letting $x \to \infty$ after appropriate estimations gives the desired result [though the calculations are more complicated than it seems at a glance!]
 
Last edited:
  • #4
[The Theory of Analytic Continuation][8]

So far so good. We have seen that more one gets closer to the singularities of $\zeta$, more the connection about primes are unveiled. We have already worked on the boundary $\Re = 1$ and revealed a great deal of information about prime numbers. It is then somewhat natural to believe that if we can somehow shift to $\{s : \Re < 1\}$ then the connection with primes would become more significant. So the question is, how to make sense of $\zeta$ in the region? This should stand as a motivation for rethinking about the complex analytic properties of $\zeta$.

It is assumed that readers of this section are familiar with fundamentals of complex analysis. If I ever have the time, I'll add it to the addendum later some time.

Let's start with an example.

$$f_1(z) = 1 + z + z^2 + z^3 + \cdots \;\;\;\;\;\;\;\;\;\;\;\;\;\;\;\;\;\;\; f_2(z) = \int_0^\infty \exp(-t(1-z)) dz$$

The domain $D_1$ of $f_1(z)$ is $\{z \in \Bbb C : |z| < 1\}$ whereas the domain $D_2$ of $f_2(z)$ is $\{z \in \Bbb C : \Re[z] < 1\}$. These two function match at the region $D_1 \cap D_2 = D_1$ of the complex plane, so the former function can be considered as a restriction of the latter. Further still, the function

$$f(z) = \frac1{1-z}$$

which is defined on $\Bbb C\setminus \{1\}$ matches with $f_2$ on the region $D_2$, so that $f_2$ can be considered as a restriction of this function.

However, note that this in no way implies that $f_1(2) = 1 + 2 + 4 + 8 + \cdots = f(2) = -1$. The domain in which they match is strictly $|z| < 1$.

Theorem : Holomorphic functions are all complex analytic, i.e., expandable in Taylor series in it's neighborhood of holomorphy.

Let $f(z)$ be a function defined on $D$ inside the complex plane, overlapped by $D_1$ and $D_2$ with intersection $D_1 \cup D_2$, which in turn overlaps $D$. Consider holomorphic functions $f_1(z)$ and $f_2(z)$ over $D_1$ and $D_2$ such that $f_1 = f$ and $f_2 = f$ throughout $D$. Consider the function $F = f_1 - f_2$ in $D_1 \cap D_2$, which is holomorphic in that domain, as both $f_1$ and $f_2$ are holomorphic in $D_1 \cap D_2$. Using the above theorem, we expand $F$ in Taylor series around some neighborhood of $c \in D$ with radius of convergence $|z-c| < R$ :

$$F(z) = f_1(z) - f_2(z) = \sum_{n = 0}^\infty a_n (z - c)^n$$

If $f_1$ and $f_2$ doesn't agree on $D_1 \cap D_2$, i.e., $F(z) \neq 0$, then there exists a $k \geq 0$ such that $a_k \neq 0$. Thus,

$$|F(z)| = |z - c|^k \left | a_k + (z - c)\cdot a_{k+1} + \cdots \right | \geq |z-c|^k \left ( |a_k| - \frac{K \cdot |z-c|}{(r-c)^{k+1}} - \frac{K \cdot |z-c|^2}{(r-c)^{k+2}} - \cdots \right )$$

For some $r$ inside the convergence disk, and a sufficiently large $K$ such that $K \geq a_n (r-c)^n$ for all $n$ (the existence is straightforward from the convergence of the series).

Note that the last sum is a geometric series, thus applying the closed form one gets

$$|F(z)| \geq |z-c|^k \left ( |a_k| - \frac{K \cdot |z-c|}{(r-c)^k(r-c-|z-c|)}\right)$$

One can choose $|z-c|$ to be sufficiently small so that the RHS is positive, which implies that $f(z)$ has no zeros inside the unit disk around $z = c$. But then $c \in D$, and $F = f_1 - f_2$ vanishes identically on $D$, so this is a contradiction. Hence the coefficients of the Taylor series of $F(z)$ must all vanish, resulting in $F(z) = 0$ and thus $f_1(z) = f_2(z)$ identically on $D_1 \cap D_2$.

The result above really proves that analytic continuations are unique in one sense. Note that this happens only for the continuation is _analytic_. There are lots of ways one can extend a function (for example, think about a beth-number extension of the Riemann $\zeta$) but it is not guaranteed that it will be unique.

Exercise : Prove that $f(z) = \sum_{n = 0}^\infty z^{3^n}$ cannot be analytically continued beyond it's convergence disk $|z| < 1$.

There, we have an excellent tool at hand to bring our dreams to life : we can finally extend $\zeta$ beyond it's convergence domain, and more than anything, extend uniquely!

Exercise : Prove that $\zeta(s)$ is analytic on the region $\Re > 1$.

We will now analytically continue $\zeta(s)$ to the left side of $\Re = 1$[6]. Firstly, note that for any function $f$ with a continuous derivative in $[a, b]$

$$\sum_{a < n \leq b} f(n) = \int_a^b f(x) \mathrm{d}x + \int_a^b \left(x - \lfloor x \rfloor -\frac12 \right) f'(x) \mathrm{d}x + \left(a - \lfloor a \rfloor - \frac12\right)f(a) - \left(b - \lfloor b \rfloor - \frac12\right)f(b)$$

The proof of pretty straightforward : since the formula is additive with respect to the interval, break $[a, b]$ up in short $[k, k+1]$ intervals. The details are left to the readers familiar with fundamentals of calculus. If one subs in $f(n) = 1/n^s$, letting $a$ and $b$ to be positive integers, then

$$\sum_{a < n \leq b} \frac1{n^s} = -s\int_a^b \frac{x - \lfloor x \rfloor - 1/2}{x^{s+1}} \mathrm{d}x - \frac{a^{1-s} - b^{1-s}}{1-s} - \frac1{2} \left ( \frac1{a^s} - \frac1{b^s} \right)$$

Letting $b \to +\infty$ and $a = 1$ results in

$$\zeta(s) = s\int_1^\infty \frac{\lfloor x \rfloor - x + 1/2}{x^{s+1}} \mathrm{d}x + \frac1{1-s} + \frac12$$

Excercise : Prove that this integral converges for $\Re > -1$, thus is an analytic continuation of $\zeta(s)$ to the extended region.

However, note that for $1 < \Re < 0$

$$\begin{align}\zeta(s) &= s\int_1^\infty \frac{\lfloor x \rfloor - x + 1/2}{x^{s+1}} \mathrm{d}x + \frac1{1-s} + \frac12 \\ &= s\int_1^\infty \frac{\lfloor x \rfloor - x + 1/2}{x^{s+1}} \mathrm{d}x + s\int_0^1 \frac{\lfloor x \rfloor - x + 1/2}{x^{s+1}} \mathrm{d}x \\ &= s\int_0^\infty \frac{\lfloor x \rfloor - x + 1/2}{x^{s+1}} \mathrm{d}x \end{align}$$

Now, from a little Fourier analysis, we see that

$$\lfloor x \rfloor - x + 1/2 = \sum_{n = 1}^\infty \frac{\sin(2n\pi x)}{n\pi}$$

Hence

$$\begin{align}\zeta(s) = s\int_1^\infty \frac{\lfloor x \rfloor - x + 1/2}{x^{s+1}} \mathrm{d}x & = s\int_1^\infty \sum_{n = 1}^\infty \frac{\sin(2n\pi x)}{x^{s+1} n\pi} \mathrm{d}x \\ &= \frac{s}{\pi} \sum_{n = 1}^\infty \frac{(2n\pi)^s}{n} \int_0^\infty \frac{\sin(t)}{t^{s+1}} \mathrm{d}t \\ &= \frac{s}{\pi} (2 \pi)^s \zeta(1 - s) \left ( -\Gamma(-s) \right)\sin(\pi s/2)\end{align}$$

This is a functional equation with reflection axis being $\Re = 1/2$. Since we already have efficient ways of computing zeta for $\Re > 0$, this functional equation can be used to produce an analytic continuation of $\zeta$ in $\Re < 0$. From this equation, we immediately find out a bunch of $\zeta$ zeros on the real axis at negative integers, coming from the factor of $\sin(2k\pi) = 0$. These points, $s = -2k$, are called trivial zeros of the zeta.

All the tools which are needed to understand the Riemann Hypothesis are finally collected throughout these 3 posts. We are now prepared to state the conjecture.
 
Last edited:
  • #5
[The RH and the music of Primes]

Statement of the conjecture : The Riemann zeta function, or rather the analytic continuation of zeta along $\Bbb C\setminus \{1\}$ has all it's nontrivial zeros at $\Re = 1/2$.

View attachment 3020

[5]$\small{Z(t) = \zeta(1/2+it) \cdot e^{i\vartheta(t)}}$

$\small {\vartheta(t) = -t/2 \cdot \log \pi + \text{arg}\, \Gamma(1/4+it/2)}$

As we have seen before, $\zeta(s)$ sprouts out a bunch of information about primes while $s$ is in the neighborhood of the cut $\Re = 1$ on the complex plane. We also hypothesized that $\zeta(s)$ (in it's series form) might sprout more information about primes if $s$ is hurled right into the undefined region of $\Re < 1$. It turns out that $\Re = 1/2$ is exactly the place where a lot of information about primes are revealed by zeta. (The region of most interest is $0 \leq \Re \leq 1$ as $\Re > 1$ is mapped onto $\Re < 1$ by the reflection formula we have derived above, while it just acts as an automorphism on the critical strip bounded by $\Re = 0$ and $\Re = 1$)

Again, it takes a little bit of complex analysis to explain the connection in full. One uses another version of the prime number theorem, which is left as an exercise for the readers :

Exercise : Define the Von Mangoldt function $\Lambda(n)$ as $\log(p)$ when $n = p^k$ for some prime $p$ and $0$ otherwise. Prove that the prime number theorem is equivalent to the identity $\sum_{n \leq x} \Lambda(n) \sim x$. This function is called the summatory Von Mangoldt function and is denoted by $\psi(x)$.

But then note that when $\Re > 1$

$$\zeta'(s) \cdot \frac{1}{\zeta(s)} = \sum_{a = 1}^\infty \frac{\log(a)}{a^s} \cdot \sum_{b = 1}^\infty \frac{\mu(b)}{b^s} = \sum_{a = 1}^\infty \sum_{b = 1}^\infty \log(a) \mu(b) \cdot \frac1{(ab)^s} \stackrel{n = ab}{=\! =} \sum_{n = 1}^\infty \left ( \sum_{ab = n} \mu(a) \log(b) \right) \frac1{n^s}$$

However,

$$\sum_{ab = n} \mu(a) \log(b) = \sum_{d|n} \mu(d) \log(n/d) = \sum_{d|n} \mu(d) \log(n) - \sum_{d|n} \mu(d) \log(d) = -\sum_{d|n} \mu(d)\log(d)$$

By fundamental theorem of arithmetic, $n$ can be written as finite product of prime powers. Since, by the definition of $\mu$, if $d$ is not square-free the particular term vanishes, we can consider without any loss of generality that $n = \Pi_i^k \, p_i$ for primes $p_i$. If that is the case, then as $\mu(p_i) = -1, \mu(p_i q_j) = 1, \cdots, \mu(n) = (-1)^k$, it thus follows that $\sum_{d|n} \mu(d)\log(d) = 0$ for composite $n$. However, if $n = p^k$ for some integer $k \geq 1$, then $\sum_{d|n} \mu(d)\log(d) = \mu(1)\log(1) + \mu(p)\log(p) + 0 = -\log(p)$. Thus, from the definition of $\Lambda(n)$, we deduce that

$$\Lambda(n) = -\sum_{d|n} \mu(d)\log(d) \Longrightarrow -\frac{\zeta'(s)}{\zeta(s)} = \sum_{n = 1}^\infty \frac{\Lambda(n)}{n^s}$$

Using Perron's formula once again with $s = 0$, we derive after appropriate estimation that for some $c > 0$, sufficiently large real $T$ and noninteger $x$

$$\int_{c-iT}^{c+iT} \frac{-\zeta'(\omega)}{\zeta(\omega)} \frac{x^\omega}{\omega} = \psi(x) + O\left(\frac{x^c}{Tc}\right) + O\left ( \frac{\log^2x}{T}\right)$$

Where $\psi(x) = \sum_{n \leq x} \Lambda(n)$ is the summatory Von Mangoldt function we have mentioned before. If now one moves the contour to the dent $[c-iT, -\Delta - iT], [-\Delta - iT, -\Delta + iT], [-\Delta + iT, c + iT]$ for some $\Delta > 1/2$, going one step ahead of what we did before, then one picks up the residue $1$ of $\zeta'(s)/\zeta(s)$ at there zeros (trivial and nontrivial) of $\zeta(s)$, $-1$ at the pole $s = 1$ and $1$ at the pole $s = 0$ of the integrand coming from the factor of $1/s$. Adding up the residues gives, after a careful estimation of the integral on the boundaries of the dents,

$$\psi(x) = x - \sum_{\rho} \frac{x^\rho}{\rho} - \frac{\zeta'(0)}{\zeta(0)} + O \left ( \frac{x\log^2 x}{T} \right) + O \left ( \frac{x^c \log^2 T}{T \log c}\right) + O \left ( \frac{\log^3 T}{\sqrt{x}}\right) + O( x^\Delta)$$

Where $\rho$ runs through all the zeta zeros in the box bounded by $\Im(z) = T$, $\Im(z) = -T$, $\Re = \Delta$ and $\Re = 1$ (trivial and nontrivial alike). Letting $\Delta \to -\infty$, $T \to \infty$ and arranging the zeta zero sum in the increasing order of $\Im(\rho)$ gives the explicit formula[9][10]

$$\psi(x) = x - \sum_{\rho} \frac{x^\rho}{\rho} - \frac{\zeta'(0)}{\zeta(0)}$$

It is immediately seen how groundbreaking this result is. The equality essentially implies (if you have finished the exercise a couple paragraphs back!) that if you know about the positions of a bunch of zeros of $\zeta$, you can compute exactly how many primes there are upto some integer.

View attachment 3021

$\small {\zeta(1/2+it)}$ in the complex plane

Intuitively, if RH is true then that should mean that we would "know" where the primes are "sitting" inside $\Bbb N$. Moreover, we can give tight estimates to the prime counting function assuming RH[9]. First, integrate the explicit formula with respect to $x$ :

$$\int_0^t \psi(x) \, \mathrm{d}x = \frac{x^2}{2} - \sum_{\rho} \frac{x^{\rho+1}}{\rho(\rho+1)} - \frac{\zeta'(0)}{\zeta(0)}x + O(1)$$

However, it takes a bit more manipulation (just manipulation though) to make this identity rigorous : we haven't proved that the zeta zero sum over there is termwise integrable.

$$\begin{align} \psi(x) \leq \int_x^{x+1} \psi(t) \, \mathrm{d}t = \int_0^{x+1} \psi(t) \, \mathrm{d}t - \int_0^x \psi(t) \, \mathrm{d}t &= \left | \frac{(x + 1)^2}{2} - \frac{x^2}{2} + \sum_\rho \frac{(x + 1)^{\rho+1} - x^{\rho+1}}{\rho(\rho+1)} + O \left ( \frac1{x^3} \right) + O(1) \right | \\ & \leq x + \left | \sum_\rho \frac{(x + 1)^{\rho+1} - x^{\rho+1}}{\rho(\rho+1)} \right | + O(1)\end{align}$$

$$\begin{align}\psi(x) \geq \int_{x - 1}^x \psi(t) \, \mathrm{d}(t) = \int_0^x \psi(t) \, \mathrm{d}t - \int_0^{x-1} \psi(t) \, \mathrm{d}t \! &= \left | \frac{x^2}{2} - \frac{(x-1)^2}{2} - \sum_\rho \frac{x^{\rho+1} - (x - 1)^{\rho+1}}{\rho(\rho+1)} + O \left ( \frac1{x^3} \right) + O(1) \right | \\ & \leq x - \left | \sum_\rho \frac{x^{\rho+1} - (x - 1)^{\rho+1}}{\rho(\rho+1)} \right | + O(1)\end{align}$$

Note that the trivial roots have already been shifted to the $O(x^{-3})$ term, and thus assuming RH $\rho = 1/2 + i \beta$ are all the possible nontrivial zeros. Using the estimates (for $|\beta| \leq x$ and $|\beta| \geq x$, respectively)

$$\left | \frac{(x + 1)^\rho - x^\rho}{\rho(\rho+1)} \right | = \frac1{|\rho|} \int_x^{x+1} t^\rho \, \mathrm{d}t \leq \frac{|x + 1|^{1/2}}{|\beta|} \leq \frac{(2x)^{1/2}}{| \beta |}$$

$$\left | \frac{(x + 1)^\rho - x^\rho}{\rho(\rho+1)} \right |\leq \frac{2|x + 1|^{\Re[\rho]+1}}{|\rho(\rho + 1)|} \leq \frac{2(2x)^{3/2}}{\beta^2}$$

One gets $\psi(x) \leq x + O(x^{1/2} \cdot \log^2 x)$ and $\psi(x) \geq x - O(x^{1/2} \cdot \log^2 x)$. These are the superbly tight bounds found by Koch[11] and up until now it's an open problem to deduce these results unconditionally.
 
Last edited:
  • #6
[A Moebius walk : Denjoy's probabilistic interpretation of RH]

Now that we have a great deal of information about the connection with zeta and primes, we will turn our head towards the proof of the theorem quoted in http://mathhelpboards.com/math-notes-49/what-riemann-hypothesis-11812.html#post55609, which, as we will see, would give rise to and interesting connection with RH

Proof of theorem :

First, let us estimate $\sum_{n \leq x} \tau(n)$. Note that $\tau(n) = \sum_{ab = n} 1$. Thus, by inclusion-exclusion principle,

$$\begin{aligned}\sum_{ab \leq x} 1 = \sum_{a \leq \sqrt{x}} \sum_{b \leq x/a} 1 + \sum_{b \leq \sqrt{x}} \sum_{a \leq x/b} 1 - \sum_{a \leq \sqrt{x}} \sum_{b \leq \sqrt{x}} 1 \, &= \sum_{a \leq \sqrt{x}} \left ( \frac{x}{a} + O(1)\right ) + \sum_{b \leq \sqrt{x}} \left ( \frac{x}{b} + O(1)\right ) - \left ( \sqrt{x} + O(1) \right )^2 \\ &= 2x \sum_{k \leq \sqrt{x}} \frac1{k} + O(\sqrt{x}) - x - O(\sqrt{x}) - O(1) \\ &= 2x \left ( \log \sqrt{x} + \gamma + O(x^{-1/2}) \right ) - x + O(\sqrt{x}) \\ & = x\log x + (2\gamma - 1)x + O(\sqrt{x})\end{aligned}$$

This trick is usually called the hyperbola method, and we will soon use a similar one to get our desired result[7]. Now consider the function

$$\Delta(x) = \sum_{n \leq x} \left ( \log n - \tau(n) + 2\gamma\right)$$

From the above estimate, combined with Stirling's estimate, we get $\Delta(x) = O(\sqrt{x})$. From $\sum_{ab = n} \mu(a)\log(b) = \Lambda(x)$ as we have proved before and $\sum_{ab = n} \mu(a) \tau(b) = 1$ which shouldn't be hard to verify from the equality $\tau(n) = \sum_{dk = n} 1$, we have

$$\sum_{ab = n} \mu(a) \sum_{n \leq x} \left ( \log b - \tau(b) + 2\gamma\right) = \sum_{n \leq x} \left ( \Lambda(n) - 1 + 2\gamma \delta_{n, 1} \right ) = \psi(x) - x + 2\gamma$$

Where $\delta_{i, j}$ is the Kronecker delta. On the other hand,

$$\sum_{ab = n} \mu(a) \sum_{n \leq x} \left ( \log b - \tau(b) + 2\gamma\right) = \sum_{b \leq K} \left ( \log k - \tau(k) + 2\gamma \right) M\left ( \frac{x}{b} \right ) + \sum_{a \leq x/K} \mu(a) \left ( \Delta(x/a) - \Delta(K)\right )$$

For some $K$ inside $[1, x]$. Assuming $M(x) = o(x)$, the first part is $o(x)$ while the last is, using $\Delta(x) = O(x)$, $O(x/\sqrt{K})$ from simple estimates. Thus as $K$ grows slowly with $x$, the last sum vanishes, leaving $\psi(x) - x = o(x)$. From a previous exercise, this implies the PNT.

Exercise : Prove that $M(x) = o(x)$ implies $\sum_{n \geq 1} \mu(n)/n = 0$, and vice-versa.

Interestingly, RH is connected to a moebius estimate too, as one might expect from the fact that PNT is really equivalent to zero-freeness at $\Re = 1$. This is derived from using the Abel summation formula with $\phi(x) = 1/x^s$, $a_n = \mu(n)$ and $A(x) = M(x)$ :

$$\sum_{n \leq x} \frac{\mu(n)}{n^s} = \frac{M(x)}{x^s} + s \int_1^x \frac{M(x) \, \mathrm{d}x}{x^{1+s}}$$

If $\Re > 1$, then as $x \to \infty$ the limit $M(x)/x^s$ vanishes as $M(x) = \sum_{n \leq x} \mu(n) \leq \sum_{n \leq x} 1 \leq x$. Thus,

$$\sum_{n \geq 1} \frac{\mu(n)}{n^s} = \frac1{\zeta(s)} = s\int_1^\infty \frac{M(x) \, \mathrm{d}x}{x^{1+s}}$$

The convergence of the integral depends upon the growth of $M(x)$. If $M(x) = O(x^\varepsilon)$ for some $\varepsilon > 0$ then the integral converges iff $\Re > \varepsilon$. Hence the integral provides an analytic continuation of $1/\zeta(s)$ to this domain. However, as $1/\zeta(s)$ has poles at $\Re = 1/2$ assuming Riemann Hypothesis, the integral cannot converge there. Thus $\varepsilon > 1/2$. Thus, a sufficient condition for RH is that $M(x) = O(x^{1/2 + \epsilon})$ for all $\epsilon > 0$. Necessity can be derived with delicate results in complex analysis, but we won't need them at this moment.

[Probability of RH]

Let a fair coin be tossed. Associate the random walk $X_n$ with the coin toss, which returns $+1$ if head is drawn and $-1$ for tail. Thus, using basics of probability theory, $E( \left | \sum_{n \leq x} X_n \right | ) \ll \sqrt{X}$. Denjoy hypothesized that $\mu(n)$, for large square-free $n$, "almost" behaves like a random variable.

https://www.physicsforums.com/attachments/3032

$\small{\mu(n)}$ for $\small{n = 1, 100}$​

That should imply that $M(x) = \sum_{n \leq x} \mu(x)$ also grows like $\sqrt{x}$, which is indeed a stronger hypothesis than $M(x) = O(x^{1/2+\epsilon})$. However, this is a false asymptotics, as Odlyzko and Riele proved in[12]

These is a slight correction of this version using Laplace limit theorem, which states that if a fair coin is flipped $N$ times then the probability that the error of the expectation of $N/2$ heads/tails is $\ll N^{1/2}$ is approximately $\int^{(2K^2/\pi)^{1/2}}_{-(2K^2/Pi)^{1/2}} \exp(-\pi x^2) \mathrm{d}x$, where $K$ is the constant of the asymptotic growth. Thus, the probability that the error is something like $\ll N^{1/2+\epsilon}$ is approximately $\int_0^{N^\epsilon (2\pi)^{1/2}} \exp(-\pi x^2) \mathrm{d}x$ which tends to $1$ as $N$ grows towards $+\infty$.

However, since $\mu(n)$ acts almost as a random variable ( i.e., the event of $i$ having $(-1)^k$ number of prime factors is expected to be independent of the event of $j$ having $(-1)^l$ number of prime factors) we can state that the number of integers smaller than $x$ having heads deviates off from $x/2$ by $x^{1/2+\epsilon}$ with probability $1$, i.e., $M(x) = O(x^{1/2 + \epsilon})$ and thus RH is true with probability $1$.

While these kinds of arguments are of probabilistic importance, in general they can't be rigorified.
 
Last edited:
  • #7
[Some computations]

We will first calculate some nontrivial zeros off-hand.

Let us introduce a very convenient function (well, it has already been mentioned below a plot) : Define the Z-function as

$$Z(t) = \exp(i \vartheta(t)) \cdot \zeta \! \left(\frac12 + it\right )$$

Where $\theta(t) = -t \log(\pi)/2 + \text{arg} \, \Gamma (1/4 + it/2 )$. The fun thing about this function is that it analytically translates the zeta function at $\Re = 1/2$ to $\Bbb R$, so you can directly study it's behavior on the real plane instead. We will first derive an approximation formula for the Riemann zeta which will help greatly in computation of the zeros. First, by residue theorem,

$$\zeta(s) - \sum_{n \leq x} \frac1{n^s} = \sum_{n > x} \frac1{n^s} = \frac1{2i} \int_{x - i\infty}^{x + i\infty} \frac{\cot \pi t}{t^s} \, \mathrm{d}t$$

Splitting the integral up in $[x - i\infty, x]$ and $[x, x + i\infty]$, one gets

$$-\frac1{2i} \int_{x - i\infty}^x \frac{\cot(\pi t) - i}{t^s} \, \mathrm{d}t - \frac1{2i} \int_x^{x + i\infty} \frac{\cot(\pi t) + i}{t^s} \, \mathrm{dt} - \frac{x^{1-s}}{1 - s}$$

The two integrals converges throughout the complex plane, thus provides an analytic continuation of the expression above. Note, however, that $|\cot(\pi t) + i| = 2(1 + \exp(2\pi r))^{-1} < 2\exp(-2\pi r)$ and $|t^{-s}| = t^{-\Re} \exp(\Im \text{arg}(t))$ $< x^{-\Re} \exp(|\Im| \theta/x)$ where $t = x + i\theta$. Thus,

$$\left | \zeta(s) - \sum_{n \leq x} \frac1{n^s} \right | \leq 2 x^{-\Re} \int_{0}^\infty \exp(-2\pi r + |\Im|\theta/x) \, \mathrm{d}r + \left | \frac{-x^{1-s}}{1-s} \right | \leq \frac{x^{1-s}}{|1-s|} + \frac{x^{-\Re}}{2\pi - |\Im| \theta/x} $$

Now multiplying out by $\exp(i\vartheta(t))$ at $s = 1/2 + it$ and taking the real part gives, for $x > t > 0$,

$$\left | Z(t) - \sum_{n \leq x} \frac{\cos(t\log n - \vartheta(t))}{n^{1/2}} \right | \leq \frac{x^{1/2}}{t} + \frac{2x^{1/2}}{2\pi x - t} $$

Picking $x = t^2$, the error transforms into $1 + 1/(\pi t - 1/2)$, which goes to $1$ as $t$ grows. For example, say we want to evaluate $Z(17)$. Using Stirling's formula, $\vartheta(t) = t/2 \cdot \log \, (t/{2\pi}) - t/2 - \pi/8 + O (1/t)$. Hence ignoring the asymptotic term, $\vartheta(17) \approx -0.43$. Running the GP snippet $\colorbox{LightGray}{$\texttt{sum(n=1,17^2,cos(17*log(n)+0.43)/sqrt(n))}$}$, one gets the approximation $Z(17) \approx 2.68377$ (which is correct with an error of $\approx 0.54$). Since $Z(0) = \zeta(1/2)$ is a negative constant, there is a sign change of $Z(t)$ on the interval $[0, 17]$. Running the snippet $\colorbox{LightGray}{$\texttt{solve(t=0,17,real(exp(I*(arg(gamma(1/4+I*t/2))-t/2*log(Pi)))*zeta(1/2+I*t)))}$}$ GP returns $t \approx 14.1347251417346937904572519$. That is precisely the imaginary part of our first zero on the 1/2-line, correct upto 24 decimal digits.

Let's get a bit serious about this and really try calculating, say, a few hundred zeros of zeta using what we have found. This takes a little bit of trickery though : we have already shown that the worst error in our approximation through that oscillating cosine series for $x = t^2$ is not more than $1$, i.e.,

$$Z(t) \sim \sum_{n \leq t^2} \frac{\cos(t\log n - \vartheta(t))}{n^{1/2}}$$

Recall that $\cos(x-k\pi) = (-1)^k \cos(x)$. So one thing we can try is finding $t_n$s such that $\vartheta(t_n) = n\pi$ (i.e., $\zeta(1/2+it)$ is real there). From our asymptotics, $Z(t)$ is expected to change sign in $[t_k, t_{k + 1}]$, so there is an expected zero of zeta with imaginary part in that interval. The GP code

$$\colorbox{LightGray}{$\begin{array}{1} \texttt{g(k)=2*Pi*exp(1)*exp(lambertw(1/exp(1)*(k+1/8)));} \\ \texttt{for(k=1,100,p=solve(x=g(k),g(k+1),real(exp(I*(arg(gamma(1/4+I*t/2))-t/2*log(Pi)))*zeta(1/2+I*t)));} \\
\texttt{print(1/2+I*p))} \end{array}$}$$

Returns the first hundred zeros of zeta

Code:
1/2 + 25.01085758014568876321379099*I
1/2 + 30.42487612585951321031189753*I
1/2 + 32.93506158773918969066236897*I
1/2 + 37.58617815882567125721776348*I
1/2 + 40.91871901214749518739812692*I
1/2 + 43.32707328091499951949612217*I
1/2 + 48.00515088116715972794247275*I
1/2 + 49.77383247767230218191678468*I
1/2 + 52.97032147771446064414729661*I
1/2 + 56.44624769706339480436775948*I
1/2 + 59.34704400260235307965364868*I
1/2 + 60.83177852460980984425990182*I
1/2 + 65.11254404808160666087505425*I
1/2 + 67.07981052949417371447882890*I
1/2 + 69.54640171117397925292685753*I
1/2 + 72.06715767448190758252210797*I
1/2 + 75.70469069908393316832691676*I
1/2 + 77.14484006887480537268266486*I
1/2 + 79.33737502024936792276359288*I
1/2 + 82.91038085408603018316483750*I
1/2 + 84.73549298051705010573531121*I
1/2 + 87.42527461312522940653166785*I
1/2 + 88.80911120763446542368234808*I
1/2 + 92.49189927055848429625972525*I
1/2 + 94.65134404051988696659792581*I
1/2 + 95.87063422824530975874102922*I
1/2 + 98.83119421819369223332442014*I
1/2 + 101.3178510057313912287854479*I
1/2 + 103.7255380404783394163984081*I
1/2 + 105.4466230523260944936708324*I
1/2 + 107.1686111842764075151233520*I
1/2 + 111.0295355431696745246564503*I
1/2 + 111.8746591769926370856120787*I
1/2 + 114.3202209154527127658909373*I
1/2 + 116.2266803208575543821608043*I
1/2 + 118.7907828659762173229791397*I
1/2 + 121.3701250024206459189455330*I
1/2 + 122.9468292935525882008174603*I
1/2 + 124.2568185543457671847320080*I
1/2 + 127.5166838795964951242793238*I
1/2 + 129.5787041999560509857680339*I
1/2 + 131.0876885309326567235663725*I
1/2 + 133.4977372029975864501304920*I
1/2 + 134.7565097533738713313260642*I
1/2 + 138.1160420545334432001915552*I
1/2 + 139.7362089521213889504500465*I
1/2 + 141.1237074040211237619403538*I
1/2 + 143.1118458076206327394051239*I
1/2 + 146.0009824867655185474025076*I
1/2 + 147.4227653425596020495211850*I
1/2 + 150.0535204207848803514324672*I
1/2 + 150.9252576122414667618525247*I
1/2 + 153.0246938111988961982565443*I
1/2 + 156.1129092942378675697501893*I
1/2 + 157.5975918175940598875305032*I
1/2 + 158.8499881714204987241749948*I
1/2 + 161.1889641375960275194373441*I
1/2 + 163.0307096871819872433110390*I
1/2 + 165.5370691879004188300389194*I
1/2 + 167.1844399781745134409577563*I
1/2 + 169.0945154155688214895058712*I
1/2 + 169.9119764794116989666998436*I
1/2 + 173.4115365195915529598461187*I
1/2 + 174.7541915233657258133787625*I
1/2 + 176.4414342977104188888926411*I
1/2 + 178.3774077760999772858309354*I
1/2 + 179.9164840202569961393400366*I
1/2 + 182.2070784843664619154070372*I
1/2 + 184.8744678483875088009606466*I
1/2 + 185.5987836777074714665277043*I
1/2 + 187.2289225835018519916415406*I
1/2 + 189.4161586560169370848522891*I
1/2 + 192.0266563607137865472836314*I
1/2 + 193.0797266038457040474022058*I
1/2 + 195.2653966795292353214631878*I
1/2 + 196.8764818409583169486222639*I
1/2 + 198.0153096762519124249199187*I
1/2 + 201.2647519437037887330161334*I
1/2 + 202.4935945141405342776866606*I
1/2 + 204.1896718031045543307164384*I
1/2 + 205.3946972021632860252123794*I
1/2 + 207.9062588878062098615019679*I
1/2 + 209.5765097168562598528356443*I
1/2 + 211.6908625953653075639074867*I
1/2 + 213.3479193597126661906391220*I
1/2 + 214.5470447834914232229442011*I
1/2 + 216.1695385082637002658695634*I
1/2 + 219.0675963490213789856772566*I
1/2 + 220.7149188393140033691155926*I
1/2 + 221.4307055546933387320974751*I
1/2 + 224.0070002546043352117288755*I
1/2 + 224.9833246695822875037825237*I
1/2 + 227.4214442796792913104614362*I
1/2 + 229.3374133055253481077600833*I
1/2 + 231.2501887004991647738061868*I
1/2 + 231.9872352531802486037716685*I
1/2 + 233.6934041789083006407044947*I
1/2 + 236.5242296658162058024755080*I
1/2 + 237.7698204809252040032366259*I
1/2 + 239.5554775733276287402689320*I

However our superiorly convenient expectation fails, just a bit after the numbers we have calculated upto. It so happens that the error (as we've seen almost always nearly $0.5$) of the asymptotics makes the zeta zero jump right outside the closure of $t_k$s. The first counterexample is $[125, 126]$ :

Code:
        gp > solve(x=g(125),g(126),real(Z(x)))
  ***   at top-level: ...lve(x=g(125),g(126),real(Z(x)))
  ***                                             ^------
  ***   domain error in solve: f(a)f(b) > 0

i.e., $Z(x)$ has no sign changes in the interval $[125, 126]$. This is the failure of Gram's law, which, had it been always true, would produce a very nice easy method for computing large zeta zeros. Fun thing is that I never knew the intuition behind Gram's law before posting all this : I was just about to calculate some very small zeros with the asymptotic formula and then it just clicked. So, well, I learned something today.


As an addendum, it'd also be fun to check the explicit formula with the data we have. Take $x = 200.1$ (not taking an integer directly for the purpose of avoiding jump discontinuities). We know that $\psi(x) = \sum_{n \leq x} \Lambda(n)$, so coding it out $\colorbox{LightGray}{$\texttt{sum(n=1,floor(200.1),-sumdiv(n, d,moebius(d)*log(d)))}$}$ returns $\approx 206.145$. Whilst making a vector $\texttt{v}$ of the zeros we computed before and running the code $\colorbox{LightGray}{$\texttt{x-sum(i=1,#v,x^(v)/v)-zeta'(0)/zeta(0)}$}$ returns real part $201.113$. The numbers are astonishingly close when $x$ and $T$ are both large numbers. For example, leaving $T = 100$, let $x=1000.1$. Then doing through the same process, one finds $\psi(x) \approx 996.68$ and for the sum real part $\approx 996.352$. Riemann's original version was directly about $\pi(x)$, and knowing enough zeros one could exactly compute $\pi(x)$ for some given large $x$. If this does not destroy the myth about having no explicit/computationally helpful formula for counting number of primes, I don't understand what one means by a "formula".
 
Last edited:
  • #8
When I was 15 or so I became very interested in the Riemann Hypothesis and the other Millennium Problems from the Clay Institute, so I bought a book called Prime Obsession that covered the RH from a historical point of view instead of a mathematical one. It is a good book for non-mathematicians that reasonably introduces concepts you can't expect an average person to know. However, this book might mislead readers into thinking that the RH is much easier to follow than it truly is since the actual proposition can be stated very succinctly. It reminds me of when I hear someone discussing String Theory with way too much confidence after watching a 30 minute TV show on The History Channel.

The summary Balarka is writing seems to be at a good level for those working at the undergrad level but isn't overwhelming to read either. If I don't follow some part I am at least aware of the topic that needs more study. I've skimmed the whole thread but will be reading in detail very soon. Well done, Balarka (Yes).
 
  • #9
Hey, Jameson, thanks very much - I am glad to hear that it was of some use. My intention at first was to use no advanced stuff like complex analysis however I soon reckoned that I couldn't ignore it. Complex analysis and number theory are intimately connected, much much more connected than real analysis is. It'd be an oversimplification of the problem if I didn't use CA.

However, the only _assumed_ result I have used is that Perron's formula. If anyone goes through the post rigorously and finds curiosity about Perron, I can prove it for them. It requires a bit contour manipulation and some hard-core estimation. In it's most general and powerful form, it is

$$\sum_{n \leq x} \frac{a_n}{n^s} = \frac1{2\pi i} \int_{c - iT}^{c + iT} \sum_{n=1}^\infty \frac{a_n}{n^{s+\omega}} \frac{x^\omega}{\omega} \mathrm{d}\omega + O \left ( \frac{x^c}{T(\sigma + c - 1)^\alpha} \right) + O \left ( \frac{f(2x) x^{1-\sigma} \log x}{T} \right ) + O \left ( \frac{f(N) x^{1-\sigma}}{T|x - N|} \right ) $$

Where $a_n = O(f(n))$, $\psi(n)$ not decreasing, $\sum_{n \leq x} |a_n|/n^\sigma = O((\sigma - 1)^{-\alpha})$ in the neighborhood of $\sigma = 1$, some $c > 0$ such that $\sigma + c > 1$ and a noninteger $x$ with $N = \text{int}(x)$.

Letting $T \to \infty$ and $s = 1$, however, one arrives at the weaker formula shown in wiki, weaker in the sense that the absolute convergence may not be verified anymore, neither of the sum nor of the integral (the zeta zero sum in the explicit formula for $\psi(x)$ a couple posts back is, for example, not absolutely converging and it'd be a capital mistake to compute the sum without ordering the roots, see Riemann's trap ;))

I've also explained some elementary version of RH, like the moebius-RH : $M(x) = O(x^{1/2+\epsilon})$ and it's probabilistic interpretation, the $\psi$-RH : $|\psi - x| \leq k \cdot \sqrt{x}^{1/2} \log^2 x$ for some $k > 0$ which is in fact the tightest bound, following Littlewood's result. I feel a relation to prime gaps (can be probabilistically explained!) would have been interesting but unfortunately I haven't studied it at all. There are a few plots and a table (and _will be_ computational results) too, so some intuition is present. In the whole, I believe the intuitive and the unintuitive part cancels each other out :D

That reminds me, I think I have to add a Venn diagram to the analytic continuation post : a lot of hubbub about analytic continuation is going round. Some are getting the wrong idea, and some never looking back at it. While analytic continuation is a delicate business in complex analysis (indeed, they are the starting point of Riemannsurfaces) I have really used nothing than basics of calculus in that post.

Finally, I'd like to have a few people reading it thoroughly and asking me anytime about anything that isn't explained well in the thread.
 
  • #10
Announcement : I have just finished the note, there will be no further posts.

The solutions are expected to be posted a few weeks later, it maybe further delayed on request from members (due to reasons like trying a problem in there, etc). Solution/ideas about any of the problems are most certainly welcome.

Pointing out any kind of typo (there are a lot of them!) are welcome, especially $\LaTeX$ ones.

Any kind of questions regarding the content will definitely be answered by the author (that's me).
 
  • #11
[Hints and Solutions]

[Problem 1.1.] Hint : For real $s$, $\sum_{n \geq 1} n^{-s}$ diverges for $s < 1$ from comparing with $n^{-1}$ termwise and recalling that the Harmonic series diverges . It converges for $s > 1$ by the condensation test. Can you use these to solve the more general problem regarding complex $s$?

[Problem 1.2.] Taking log of the product, $-\log \prod_{p} (1 - 1/p^s) = -\sum_p \log(1 - 1/p^s) = \sum_p 1/p^s + O(1)$ and comparing with $\sum_{n \leq 1} 1/n^s$ for $\Re > 1$ proves that the product converges regardless of infinitude of primes.

[Problem 1.3.] $\zeta(2) = \pi^2/6$, and by the product formula, $\pi^2/6 = \prod (1-1/p^s)^{-1}$. If there were finitely many primes, the RHS would be rational but then $\pi^2$ is irrational, a contradiction. Thus there are infinitely many primes.

[Problem 2.1.] From the PNT, probability that two random numbers chosen on $[1, x]$ are both primes is about $1/\log(x) \cdot 1/\log(x) = 1/\log^2(x)$. So assuming that the events of $p$ and $p+2$ being primes are independent, probability that they are both primes is $1/\log^2(x)$. Thus it is expected that $\pi_2(x) \sim \int_2^x dt/\log^2(t) $. However, it is unfair to expect that the events of $p$ and $p + 2$ being primes are independent. Indeed, if $p$ is prime, then the other is odd which gives doubles the probability of it being prime w.r.t. probability of being a truly random number being prime. So that's a correction factor of $2$. Further, for large prime $p$ to be prime, $p$ must not be divisible by primes $q$ below it. The number of residue classes of $p$ modulo $q$ is $\phi(q) = q - 1$, and there are precisely $q - 1 - 1 = q-2$ residue classes for which $q \not | p + 2$. Thus, probability that $p+2$ is not divisible by $q$ too is $(q - 2)/(q - 1)$. Similarly, for a truly random number the probability that it's not divisible by $q$ is clearly $(q - 1)/q$. The second correction factor is then about $\prod_q (q-2)/(q-1) \cdot q/(q-1) = \prod_q (1 - 1/(q-1)^2)$. Hence we have the desired. [this product converges to a constant called the twin prime constant and is denoted by $C_2$]

[Problem 3.1.] $f(z) = z + z^3 + z^9 + z^{27} + \cdots$. Note that $f$ has a singularity at $z = 1$. Rearranging, we see that $f(z) = z + f(z^3)$. At the points $z^3 = 1$, $f(z^3)$ have singularities, hence so does $f(z)$. Further, $f(z^3) = z^3 + f(z^9)$, so $f(z)$ also has singularities at $z^9 = 1$. In particular, $f(z)$ has singularities on the points $z_0$ satisfying ${z_0}^{3^n} = 1$ for any integer $n$. These points are dense on the unit circle, so $f(z)$ has singularities on all of the unit disc $|z| = 1$, thus it cannot be analytically continued outside it's domain.

[Problem 3.2.] Weierstrass M-test guarantees that $\zeta(s)$ not only converges but converges uniformly in $\Re > 1$. Note that $1/n^s$ is analytic for any positive nonzero integers $n$. By Cauchy's theorem, $\oint_C 1/n^s \, \mathrm{d}s = 0$ for any simple closed contour $C$. $\oint_C \zeta(s) = \oint_C \sum_{n \geq 1} 1/n^s \, \mathrm{d}s = \sum_{n \geq 1} \oint_C 1/n^s \, \mathrm{d}s = 0$ where interchange of the sum and integral is justified regardless of $C$ from the uniform convergence. By Morera's theorem, $\zeta(s)$ is analytic on $\Re > 1$.

[Problem 3.3] Hint : $1/2-\{x\}$ is bounded everywhere on $\Bbb R$. Note that $\int_1^\infty x^{-s-1} \, \mathrm{d}x$ converges iff $s + 1 > 0$.

[Problem 4.1] Note that $\psi(x) = \sum_{p^k \leq x} \log p$ where $p$ runs through primes. The term $\log q$ appears $k$ times on the sum, where $k$ is the number of $q$-powers appearing below $x$. $q^k \leq x$ implies $k \leq \log(x)/\log(p)$ hence the maximal number of appearance of $q$-powers below $x$ is $\lfloor \log(x)/\log(p) \rfloor$. Thus, $\psi(x) = \sum_{p \leq x} \log(p) \cdot \lfloor \log(x)/\log(p) \rfloor \leq \sum_{p \leq x} \log(x) = \pi(x) \log(x)$. Similarly, it can be shown that $\psi(x) \geq (1 - \epsilon) \pi(x) \log(x) + (1 - \epsilon) \log(x) \cdot O(x^{1-\epsilon})$ for any $\epsilon > 0$. Thus $\psi(x)/x$ is squeezed between $\pi(x)\log(x)/x$ and $(1 - \epsilon)\pi(x)\log(x)/x$, hence the PNT implies $\lim_{x \to \infty} \psi(x)/x = 1$ as $\epsilon \to 0$ and vice versa.

[Problem 5.1] $n \leq x$ implies $1/n \geq 1/x$, hence $\sum_{n \leq x} \mu(n)/n \geq 1/x \sum_{n \leq x} \mu(n) = M(x)/x$ which, by out hypothesis, is $o(1)$. On the other hand, Abel summation results $\sum_{n \leq x} \mu(n)/n = M(x)/x + \int_1^x M(t)/t^2 \, \mathrm{d}t \leq M(x)/x = o(1)$. Squeezed between $o(1)$s, as $x \to \infty$, $\sum_{n \leq x} \mu(n)/n \to 0$.
 
Last edited:

1. What is the Riemann Hypothesis?

The Riemann Hypothesis is a famous unsolved mathematical problem that was first proposed by German mathematician Bernhard Riemann in 1859. It deals with the distribution of prime numbers and their connection to the complex number system.

2. Why is the Riemann Hypothesis important?

The Riemann Hypothesis is important because it has far-reaching implications in many areas of mathematics and science, including number theory, physics, and cryptography. Its proof would also provide a deeper understanding of the nature of prime numbers and their behavior.

3. What is the significance of solving the Riemann Hypothesis?

Solving the Riemann Hypothesis would not only be a major achievement in mathematics, but it could also have practical applications in fields such as cryptography, where prime numbers are used to secure data and communications.

4. How many attempts have been made to prove the Riemann Hypothesis?

There have been countless attempts to prove the Riemann Hypothesis over the past 160 years, but no one has been able to provide a conclusive proof. Many mathematicians have made progress in understanding and providing evidence for the hypothesis, but it still remains unsolved.

5. What are some potential consequences if the Riemann Hypothesis is proven to be false?

If the Riemann Hypothesis is proven to be false, it would have a significant impact on our current understanding of prime numbers and their distribution. It would also raise questions about the validity of other mathematical theories that have been built upon the assumption of the Riemann Hypothesis being true.

Similar threads

  • Linear and Abstract Algebra
Replies
1
Views
4K
  • Linear and Abstract Algebra
Replies
4
Views
7K
  • Linear and Abstract Algebra
Replies
6
Views
6K
  • General Discussion
Replies
2
Views
920
  • Sci-Fi Writing and World Building
Replies
12
Views
3K
  • Beyond the Standard Models
Replies
2
Views
2K
Replies
1
Views
1K
  • Quantum Interpretations and Foundations
Replies
25
Views
1K
  • Astronomy and Astrophysics
Replies
2
Views
1K
Replies
46
Views
9K
Back
Top